Wie kann man beweisen, dass Quanten N=4N=4{\cal N}=4 Super-Yang-Mühlen superkonform sind?

Ich interessiere mich besonders für elegante, leuchtende Beweise, die nicht viele einfache technische Berechnungen erfordern

Gibt es auch einen störungsfreien Beweis?

Betrachten Sie es als eine N=1-Super-Yang-Mills-Theorie mit drei angrenzenden chiralen Superfeldern und wenden Sie die nicht-perturbative Analyse von Leigh-Strassler an .

Antworten (1)

In jeder supersymmetrischen Theorie können Sie die Eichkopplung als Koeffizient von wählen W a 2 im Superpotential. Diese Eichkopplung läuft nur bei einer Schleife, was eine grundlegende Folge der Nicht-Renormierungssätze ist. Die anderen möglichen Laufkoeffizienten sind die kinetischen Terme, Z ( μ ) Q Q . Diese werden in der Störungstheorie im Allgemeinen auf alle Ordnungen renormiert.

In N = 4 der Ein-Schleifen-Koeffizient ist Null. Dies ist trivial (nur die Felder zählen). Daher läuft die Messkupplung (wie oben definiert) nicht. Aber N = 4 verbindet die Eichteilchen mit den chiralen Superfeldern (alle Materieteilchen sitzen in einer großen Darstellung von N = 4 ) und kann daher auch nicht renormiert werden.

Dies ist ein geschicktes und intuitives Argument ... Eine ähnliche Logik funktioniert in vielen N = 2 auch Theorien.

Gute Antwort. Können Sie bitte eine Referenz angeben? Verstehe ich richtig, dass dieses Argument nur störend ist? Wenn ja, gibt es eine Möglichkeit, es auf ein nicht störungsfreies zu erweitern?
Sie ist störungsfrei, weil die Messgerätekopplung nicht störungsfrei läuft. Ich habe es nirgendwo explizit geschrieben gesehen, aber ich bin sicher, dass ich (bei weitem) nicht der erste bin, der diesen Gedanken hatte :)
Dein Argument ist definitiv schöner :)
Yuji: Ich denke, Sie müssen trotzdem auf dieses Argument zurückgreifen, selbst wenn Sie Leigh-Strassler tun. Dies liegt daran, dass letzteres nur eine eindimensionale Linie von CFTs impliziert, aber nicht beweist, dass diese eindimensionale Linie mit der Linie übereinstimmt, auf der N = 4 sitzt. Irgendwann muss man also die höhere Symmetrie aufrufen.
Ok, können Sie also einen Hinweis auf den von Ihnen verwendeten Nicht-Renormalisierungssatz geben?
Seite 96 in physical.uc.edu/~argyres/661/susy1996.pdf überprüft das Argument. Im Allgemeinen können Sie beweisen, dass es sich um Beiträge mit einer Schleife und nicht störenden handelt. Da es in N=4 keine Einschleife gibt, gibt es kein \Lambda und somit fehlen auch die nicht-störenden Beiträge.
Ja, das ist richtig. Der Geschmack der Argumentation dort scheint dem, was ich gesagt habe, sehr ähnlich zu sein. Ich gehe davon aus, dass ein ähnlicher Ansatz für Theorien in 3D oder allgemeiner für jede Theorie verwendet werden kann, bei der der Begriff der Holomorphie sinnvoll ist.